Convergence in distribution of Poisson variables.












1














Consider ${ X_{i} }$ are independent random variables with Poisson distribution. We want to know about convergence in distribution of $frac{S_{n}-mathbb{E}(S_{n})}{sqrt{operatorname{Var}(S_{n})}}$.



There are two cases :



Case 1 : $sum_{i}lambda_{i}$ converges. Then we can say variance and mean value of $S_{n}$ converges to some fixed value. Also we know that sum of Poisson random variables is also the Poisson random variable with parameter equals sum of previous ones.
So in my opinion $frac{S_{n}-mathbb{E}(S_{n})}{sqrt{operatorname{Var}(S_{n})}}$ converges to $frac{mathrm{Pois}(c) - c}{sqrt{c}}$ , where $c = sum_{i} lambda_{i}$



Case 2 : we have that $sum_{i} lambda_{i}$ diverges. Then I guess we can try to use Central limit theorem and satisfy that $frac{S_{n}-mathbb{E}(S_{n})}{sqrt{operatorname{Var}(S_{n})}}$ converges to $N(0,1)$



Am I right ? Or where have I problems to fix ?










share|cite|improve this question




















  • 1




    How do you expand "i.i.d."? In standard terminology if $X_i$'s are i.i.d Poisson then the parameters $lambda_i$ are all the same, so CLT applies.
    – Kavi Rama Murthy
    May 11 at 8:04










  • @KaviRamaMurthy my bad!
    – openspace
    May 11 at 8:04










  • @KaviRamaMurthy changed
    – openspace
    May 11 at 8:15
















1














Consider ${ X_{i} }$ are independent random variables with Poisson distribution. We want to know about convergence in distribution of $frac{S_{n}-mathbb{E}(S_{n})}{sqrt{operatorname{Var}(S_{n})}}$.



There are two cases :



Case 1 : $sum_{i}lambda_{i}$ converges. Then we can say variance and mean value of $S_{n}$ converges to some fixed value. Also we know that sum of Poisson random variables is also the Poisson random variable with parameter equals sum of previous ones.
So in my opinion $frac{S_{n}-mathbb{E}(S_{n})}{sqrt{operatorname{Var}(S_{n})}}$ converges to $frac{mathrm{Pois}(c) - c}{sqrt{c}}$ , where $c = sum_{i} lambda_{i}$



Case 2 : we have that $sum_{i} lambda_{i}$ diverges. Then I guess we can try to use Central limit theorem and satisfy that $frac{S_{n}-mathbb{E}(S_{n})}{sqrt{operatorname{Var}(S_{n})}}$ converges to $N(0,1)$



Am I right ? Or where have I problems to fix ?










share|cite|improve this question




















  • 1




    How do you expand "i.i.d."? In standard terminology if $X_i$'s are i.i.d Poisson then the parameters $lambda_i$ are all the same, so CLT applies.
    – Kavi Rama Murthy
    May 11 at 8:04










  • @KaviRamaMurthy my bad!
    – openspace
    May 11 at 8:04










  • @KaviRamaMurthy changed
    – openspace
    May 11 at 8:15














1












1








1







Consider ${ X_{i} }$ are independent random variables with Poisson distribution. We want to know about convergence in distribution of $frac{S_{n}-mathbb{E}(S_{n})}{sqrt{operatorname{Var}(S_{n})}}$.



There are two cases :



Case 1 : $sum_{i}lambda_{i}$ converges. Then we can say variance and mean value of $S_{n}$ converges to some fixed value. Also we know that sum of Poisson random variables is also the Poisson random variable with parameter equals sum of previous ones.
So in my opinion $frac{S_{n}-mathbb{E}(S_{n})}{sqrt{operatorname{Var}(S_{n})}}$ converges to $frac{mathrm{Pois}(c) - c}{sqrt{c}}$ , where $c = sum_{i} lambda_{i}$



Case 2 : we have that $sum_{i} lambda_{i}$ diverges. Then I guess we can try to use Central limit theorem and satisfy that $frac{S_{n}-mathbb{E}(S_{n})}{sqrt{operatorname{Var}(S_{n})}}$ converges to $N(0,1)$



Am I right ? Or where have I problems to fix ?










share|cite|improve this question















Consider ${ X_{i} }$ are independent random variables with Poisson distribution. We want to know about convergence in distribution of $frac{S_{n}-mathbb{E}(S_{n})}{sqrt{operatorname{Var}(S_{n})}}$.



There are two cases :



Case 1 : $sum_{i}lambda_{i}$ converges. Then we can say variance and mean value of $S_{n}$ converges to some fixed value. Also we know that sum of Poisson random variables is also the Poisson random variable with parameter equals sum of previous ones.
So in my opinion $frac{S_{n}-mathbb{E}(S_{n})}{sqrt{operatorname{Var}(S_{n})}}$ converges to $frac{mathrm{Pois}(c) - c}{sqrt{c}}$ , where $c = sum_{i} lambda_{i}$



Case 2 : we have that $sum_{i} lambda_{i}$ diverges. Then I guess we can try to use Central limit theorem and satisfy that $frac{S_{n}-mathbb{E}(S_{n})}{sqrt{operatorname{Var}(S_{n})}}$ converges to $N(0,1)$



Am I right ? Or where have I problems to fix ?







probability-theory proof-verification weak-convergence poisson-distribution probability-limit-theorems






share|cite|improve this question















share|cite|improve this question













share|cite|improve this question




share|cite|improve this question








edited Nov 24 at 22:52









Davide Giraudo

125k16150259




125k16150259










asked May 11 at 7:58









openspace

3,4452822




3,4452822








  • 1




    How do you expand "i.i.d."? In standard terminology if $X_i$'s are i.i.d Poisson then the parameters $lambda_i$ are all the same, so CLT applies.
    – Kavi Rama Murthy
    May 11 at 8:04










  • @KaviRamaMurthy my bad!
    – openspace
    May 11 at 8:04










  • @KaviRamaMurthy changed
    – openspace
    May 11 at 8:15














  • 1




    How do you expand "i.i.d."? In standard terminology if $X_i$'s are i.i.d Poisson then the parameters $lambda_i$ are all the same, so CLT applies.
    – Kavi Rama Murthy
    May 11 at 8:04










  • @KaviRamaMurthy my bad!
    – openspace
    May 11 at 8:04










  • @KaviRamaMurthy changed
    – openspace
    May 11 at 8:15








1




1




How do you expand "i.i.d."? In standard terminology if $X_i$'s are i.i.d Poisson then the parameters $lambda_i$ are all the same, so CLT applies.
– Kavi Rama Murthy
May 11 at 8:04




How do you expand "i.i.d."? In standard terminology if $X_i$'s are i.i.d Poisson then the parameters $lambda_i$ are all the same, so CLT applies.
– Kavi Rama Murthy
May 11 at 8:04












@KaviRamaMurthy my bad!
– openspace
May 11 at 8:04




@KaviRamaMurthy my bad!
– openspace
May 11 at 8:04












@KaviRamaMurthy changed
– openspace
May 11 at 8:15




@KaviRamaMurthy changed
– openspace
May 11 at 8:15










1 Answer
1






active

oldest

votes


















0














The answer is easy if you compute the characteristic function explicitly. Let $u_n=lambda_1+lambda_2++...+lambda_n$. Then $Ee^{it(S_n-ES_n)/sqrt (Var(S_n)}=e^{-itsqrt (u_n)}prod_{j=1}^{n}(Ee^{itX_j /sqrt(u_n)}$. The Poisson characteristic function (paramater $lambda $) is $e^{-lambda (1- e^{it})}$. We get $e^{-itsqrt (u_n)}e^{-u_n (1-e^{it/sqrt (u_n)})}$. Using the Taylor expansion of $e^{it/sqrt (u_n)}$ up to the term in $t^{2}$ you will see that that characteristic function indeed converges to $e^{-t^{2}/2}$ if $sum lambda_j =infty$. The characteristic function also converges when $sum lambda_j <infty$ and you can write down the limiting characteristic function explictly.






share|cite|improve this answer





















    Your Answer





    StackExchange.ifUsing("editor", function () {
    return StackExchange.using("mathjaxEditing", function () {
    StackExchange.MarkdownEditor.creationCallbacks.add(function (editor, postfix) {
    StackExchange.mathjaxEditing.prepareWmdForMathJax(editor, postfix, [["$", "$"], ["\\(","\\)"]]);
    });
    });
    }, "mathjax-editing");

    StackExchange.ready(function() {
    var channelOptions = {
    tags: "".split(" "),
    id: "69"
    };
    initTagRenderer("".split(" "), "".split(" "), channelOptions);

    StackExchange.using("externalEditor", function() {
    // Have to fire editor after snippets, if snippets enabled
    if (StackExchange.settings.snippets.snippetsEnabled) {
    StackExchange.using("snippets", function() {
    createEditor();
    });
    }
    else {
    createEditor();
    }
    });

    function createEditor() {
    StackExchange.prepareEditor({
    heartbeatType: 'answer',
    autoActivateHeartbeat: false,
    convertImagesToLinks: true,
    noModals: true,
    showLowRepImageUploadWarning: true,
    reputationToPostImages: 10,
    bindNavPrevention: true,
    postfix: "",
    imageUploader: {
    brandingHtml: "Powered by u003ca class="icon-imgur-white" href="https://imgur.com/"u003eu003c/au003e",
    contentPolicyHtml: "User contributions licensed under u003ca href="https://creativecommons.org/licenses/by-sa/3.0/"u003ecc by-sa 3.0 with attribution requiredu003c/au003e u003ca href="https://stackoverflow.com/legal/content-policy"u003e(content policy)u003c/au003e",
    allowUrls: true
    },
    noCode: true, onDemand: true,
    discardSelector: ".discard-answer"
    ,immediatelyShowMarkdownHelp:true
    });


    }
    });














    draft saved

    draft discarded


















    StackExchange.ready(
    function () {
    StackExchange.openid.initPostLogin('.new-post-login', 'https%3a%2f%2fmath.stackexchange.com%2fquestions%2f2776301%2fconvergence-in-distribution-of-poisson-variables%23new-answer', 'question_page');
    }
    );

    Post as a guest















    Required, but never shown

























    1 Answer
    1






    active

    oldest

    votes








    1 Answer
    1






    active

    oldest

    votes









    active

    oldest

    votes






    active

    oldest

    votes









    0














    The answer is easy if you compute the characteristic function explicitly. Let $u_n=lambda_1+lambda_2++...+lambda_n$. Then $Ee^{it(S_n-ES_n)/sqrt (Var(S_n)}=e^{-itsqrt (u_n)}prod_{j=1}^{n}(Ee^{itX_j /sqrt(u_n)}$. The Poisson characteristic function (paramater $lambda $) is $e^{-lambda (1- e^{it})}$. We get $e^{-itsqrt (u_n)}e^{-u_n (1-e^{it/sqrt (u_n)})}$. Using the Taylor expansion of $e^{it/sqrt (u_n)}$ up to the term in $t^{2}$ you will see that that characteristic function indeed converges to $e^{-t^{2}/2}$ if $sum lambda_j =infty$. The characteristic function also converges when $sum lambda_j <infty$ and you can write down the limiting characteristic function explictly.






    share|cite|improve this answer


























      0














      The answer is easy if you compute the characteristic function explicitly. Let $u_n=lambda_1+lambda_2++...+lambda_n$. Then $Ee^{it(S_n-ES_n)/sqrt (Var(S_n)}=e^{-itsqrt (u_n)}prod_{j=1}^{n}(Ee^{itX_j /sqrt(u_n)}$. The Poisson characteristic function (paramater $lambda $) is $e^{-lambda (1- e^{it})}$. We get $e^{-itsqrt (u_n)}e^{-u_n (1-e^{it/sqrt (u_n)})}$. Using the Taylor expansion of $e^{it/sqrt (u_n)}$ up to the term in $t^{2}$ you will see that that characteristic function indeed converges to $e^{-t^{2}/2}$ if $sum lambda_j =infty$. The characteristic function also converges when $sum lambda_j <infty$ and you can write down the limiting characteristic function explictly.






      share|cite|improve this answer
























        0












        0








        0






        The answer is easy if you compute the characteristic function explicitly. Let $u_n=lambda_1+lambda_2++...+lambda_n$. Then $Ee^{it(S_n-ES_n)/sqrt (Var(S_n)}=e^{-itsqrt (u_n)}prod_{j=1}^{n}(Ee^{itX_j /sqrt(u_n)}$. The Poisson characteristic function (paramater $lambda $) is $e^{-lambda (1- e^{it})}$. We get $e^{-itsqrt (u_n)}e^{-u_n (1-e^{it/sqrt (u_n)})}$. Using the Taylor expansion of $e^{it/sqrt (u_n)}$ up to the term in $t^{2}$ you will see that that characteristic function indeed converges to $e^{-t^{2}/2}$ if $sum lambda_j =infty$. The characteristic function also converges when $sum lambda_j <infty$ and you can write down the limiting characteristic function explictly.






        share|cite|improve this answer












        The answer is easy if you compute the characteristic function explicitly. Let $u_n=lambda_1+lambda_2++...+lambda_n$. Then $Ee^{it(S_n-ES_n)/sqrt (Var(S_n)}=e^{-itsqrt (u_n)}prod_{j=1}^{n}(Ee^{itX_j /sqrt(u_n)}$. The Poisson characteristic function (paramater $lambda $) is $e^{-lambda (1- e^{it})}$. We get $e^{-itsqrt (u_n)}e^{-u_n (1-e^{it/sqrt (u_n)})}$. Using the Taylor expansion of $e^{it/sqrt (u_n)}$ up to the term in $t^{2}$ you will see that that characteristic function indeed converges to $e^{-t^{2}/2}$ if $sum lambda_j =infty$. The characteristic function also converges when $sum lambda_j <infty$ and you can write down the limiting characteristic function explictly.







        share|cite|improve this answer












        share|cite|improve this answer



        share|cite|improve this answer










        answered May 11 at 9:22









        Kavi Rama Murthy

        49.9k31854




        49.9k31854






























            draft saved

            draft discarded




















































            Thanks for contributing an answer to Mathematics Stack Exchange!


            • Please be sure to answer the question. Provide details and share your research!

            But avoid



            • Asking for help, clarification, or responding to other answers.

            • Making statements based on opinion; back them up with references or personal experience.


            Use MathJax to format equations. MathJax reference.


            To learn more, see our tips on writing great answers.





            Some of your past answers have not been well-received, and you're in danger of being blocked from answering.


            Please pay close attention to the following guidance:


            • Please be sure to answer the question. Provide details and share your research!

            But avoid



            • Asking for help, clarification, or responding to other answers.

            • Making statements based on opinion; back them up with references or personal experience.


            To learn more, see our tips on writing great answers.




            draft saved


            draft discarded














            StackExchange.ready(
            function () {
            StackExchange.openid.initPostLogin('.new-post-login', 'https%3a%2f%2fmath.stackexchange.com%2fquestions%2f2776301%2fconvergence-in-distribution-of-poisson-variables%23new-answer', 'question_page');
            }
            );

            Post as a guest















            Required, but never shown





















































            Required, but never shown














            Required, but never shown












            Required, but never shown







            Required, but never shown

































            Required, but never shown














            Required, but never shown












            Required, but never shown







            Required, but never shown







            Popular posts from this blog

            Plaza Victoria

            In PowerPoint, is there a keyboard shortcut for bulleted / numbered list?

            How to put 3 figures in Latex with 2 figures side by side and 1 below these side by side images but in...